LSAT and Law School Admissions Forum

Get expert LSAT preparation and law school admissions advice from PowerScore Test Preparation.

User avatar
 Dave Killoran
PowerScore Staff
  • PowerScore Staff
  • Posts: 5853
  • Joined: Mar 25, 2011
|
#46608
Complete Question Explanation
(The complete setup for this game can be found here: lsat/viewtopic.php?t=17159)

The correct answer choice is (D)

Answer choice (A) is incorrect because from the fourth rule if M is selected, W must be selected.

Answer choice (B) is incorrect because from the first rule at least two topazes must be selected.

Answer choice (C) is incorrect because two sapphires are selected (J and K) so only one ruby can be selected (but both F and G are selected here).

Answer choice (D) is the correct answer choice.

Answer choice (E) is incorrect because from the third rule W and Z cannot be selected together.

Get the most out of your LSAT Prep Plus subscription.

Analyze and track your performance with our Testing and Analytics Package.